Ejercicio sobre inducción matemática con sumatorias

Tengo este ejercicio pero no logro resolverlo:

Si

$$\begin{align}&x_1,x_2,...,x_n\end{align}$$

son números positivos y si

$$\begin{align}&y_k = \frac{1}{x_k}\end{align}$$

demostrar que:

$$\begin{align}&( \displaystyle\sum_{k=1}^n x_k)*(	 \displaystyle\sum_{k=1}^n y_k)   \ge n^2 \end{align}$$
Respuesta
2

Vamos a intentar resolverlo...

$$\begin{align}&Planteo\\&\bigg(\sum_{k=1}^{n}x_k\bigg) \cdot \bigg(\sum_{k=1}^{n}y_k\bigg) \ge n^2\\&\text{Que escrito de otro modo, podría ser}\\&\bigg(\sum_{k=1}^{n}x_k\bigg) \cdot \bigg(\sum_{k=1}^{n}\frac{1}{x_k} \bigg) \ge n^2\\&\text{Ahora sí, intentemos demostrarlo por inducción}\\&Caso \ base (n=1)\\&\bigg(\sum_{k=1}^{1}x_k\bigg) \cdot \bigg(\sum_{k=1}^{1}\frac{1}{x_k} \bigg)=x_1\cdot \frac{1}{x_1} = 1  \ge 1^2 (Cumple)\\&Paso \ inductivo \ (P(n)  \to^? P(n+1))\\&\bigg[ \bigg(\sum_{k=1}^{n}x_k\bigg) \cdot \bigg(\sum_{k=1}^{n}\frac{1}{x_k} \bigg) \ge n^2\bigg] \to ^? \bigg[ \bigg(\sum_{k=1}^{n+1}x_k\bigg) \cdot \bigg(\sum_{k=1}^{n+1}\frac{1}{x_k} \bigg) \ge (n+1)^2\bigg] \\&Veamos...\\&\sum_{k=1}^{n+1}x_k \cdot \sum_{k=1}^{n+1}\frac{1}{x_k}  = \bigg(\sum_{k=1}^{n}x_k + x_{n+1}\bigg) \cdot \bigg(\sum_{k=1}^{n}\frac{1}{x_k} + \frac{1}{x_{n+1}}\bigg) = \\&\sum_{k=1}^{n}x_k \sum_{k=1}^{n}\frac{1}{x_k} + x_{n+1}  \sum_{k=1}^{n}\frac{1}{x_k} + \frac{1}{x_{n+1}}\sum_{k=1}^{n}x_k + x_{n+1}\frac{1}{x_{n+1}} = \\&\sum_{k=1}^{n}x_k \sum_{k=1}^{n}\frac{1}{x_k} + x_{n+1}  \sum_{k=1}^{n}\frac{1}{x_k} + \frac{1}{x_{n+1}}\sum_{k=1}^{n}x_k + 1 \ge (Hipotesis)\\&n^2 + x_{n+1}  \sum_{k=1}^{n}\frac{1}{x_k} + \frac{1}{x_{n+1}}\sum_{k=1}^{n}x_k + 1 \ge ^? (n+1)^2\\&\text{Para demostrar esta última desigualdad y que valga la inducción, desarrollamos el binomio de la derecha y nos queda demostrar que}\\& x_{n+1}  \sum_{k=1}^{n}\frac{1}{x_k} + \frac{1}{x_{n+1}}\sum_{k=1}^{n}x_k  \ge  2n\\&\text{Antes de seguir voy a hacer 2 supuestos (\sin perder generalidad)}\\&\text{1. Hay al menos 2 valores (el caso donde n=1 ya lo probamos como caso base)}\\&\text{Sabemos que los valores son todos positivos, además voy a suponer que están ordenados de la seguiente manera: }\\&x_1 \le x_2 \le...\le x_n\\&\text{Si este no fuera el caso, podríamos considerar una nueva sucesión con los mismos valores pero ordenados, así que no perdemos generalidad con esto}\\&\text{Ahora sí, sigamos con la demostración}\\& x_{n+1}  \sum_{k=1}^{n}\frac{1}{x_k} + \frac{1}{x_{n+1}}\sum_{k=1}^{n}x_k  =\\&\bigg(\frac{x_{n+1}}{x_1} + \frac{x_{n+1}}{x_2}+ ...+ \frac{x_{n+1}}{x_n} \bigg)+ \bigg(\frac{x_{1}}{x_{n+1}} + \frac{x_{2}}{x_{n+1}}+ ...+ \frac{x_{n}}{x_{n+1}} \bigg)=\\&\text{de la forma que construimos los valores los primero paréntesis tienen todos valores mayores que 1 (o iguales), }\\&\text{mientras que el segundo paréntesis tiene todos términos menores (o iguales) a 1}\end{align}$$

Hasta acá llegué por ahora, lo dejo hasta ahí por si te sirve y podés avanzar, sino intentaré volver más tarde (creo que se puede volver a intentar con inducción con la última parte que quedó (pero con el caso base cuando n=2)

Salu2

Muchas gracias de verdad, podrías terminarlo tú para verificar?

Paul, lamentablemente mis conocimientos matemáticos, no alcanzan para esta última demostración, aunque estoy casi seguro que no se puede resolver por inducción y lo que hay que hacer es separarlo en casos. Creo que de los que formamos parte de esta página, el más capacitado para esto es el profe Valero Angel Serrano Mercadal, pero lamentablemente por temas con los administradores de la página últimamente no está participando demasiado.

Profe, en caso que tenga ganas :-), de ser posible le pido si puede terminar la demostración que hice, ya que para que la demostración esté completa queda demostrar una desigualdad.

1 respuesta más de otro experto

Respuesta
3

H o l a Paul y Gustavo.

Según dice Gustavo hay que demostrar:

$$\begin{align}&x_{n+1}  \sum_{k=1}^{n}\frac{1}{x_k} + \frac{1}{x_{n+1}}\sum_{k=1}^{n}x_k\ge 2n\\&\\&\text{veámoslo}\\&\\&x_{n+1}  \sum_{k=1}^{n}\frac{1}{x_k} + \frac{1}{x_{n+1}}\sum_{k=1}^{n}x_k=\\&\\&\sum_{k=1}^n \frac{x_{n+1}}{x_k}+\sum_{k=1}^{n}\frac{ x_k}{ x_{n+1}} =\\&\\&\sum_{k=1}^n \left(\frac{x_{n+1}}{x_k}+\frac{x_k}{x_{n+1}}  \right)=\sum_{k=1}^{n} \frac{x_{n+1}^2+x_k^2}{x_k·x_{n+1}}=\\&\\&\sum_{k=1}^n \frac{(x_{n+1}+x_k)^2-2x_{n+1}·x_k}{x_k·x_{n+1}}=\\&\\&\sum_{k=1}^n \left( \frac{(x_{n+1}+x_k)^2}{x_k·x_{n+1}}-2 \right)=\\&\\&\sum_{k=1}^n \left( \left(\frac{x_{n+1}+x_k}{\sqrt{x_k·x_{n+1}}}\right)^2-2 \right)=\\&\\&\sum_{k=1}^n \left( 4·\left(\frac{\frac{x_{n+1}+x_k}{2}}{\sqrt{x_k·x_{n+1}}}\right)^2-2 \right)\ge\\&\\&\text{La media aritmética es mayor o igual que la geométrica}\\&\text{Por lo tanto su cociente es mayor o igual que 1}\\&\\&\sum_{k=1}^n \left( 4·1^2-2 \right)=\sum_{k=1}^n 2=2n\\&\\&\text{luego resumiendo}\\&\\&x_{n+1}  \sum_{k=1}^{n}\frac{1}{x_k} + \frac{1}{x_{n+1}}\sum_{k=1}^{n}x_k\ge 2n\\&\\&\end{align}$$

Y con esto y lo que había hecho Gustavo antes queda demostrado.

S a l u d o s.

Añade tu respuesta

Haz clic para o

Más respuestas relacionadas